Difference between revisions of "1991 AHSME Problems/Problem 30"

m
m (Solution)
Line 6: Line 6:
  
 
== Solution ==
 
== Solution ==
<math>\fbox{}</math>
+
<math>\fbox{B}</math>
  
 
== See also ==
 
== See also ==

Revision as of 15:58, 28 September 2014

Problem

For any set $S$, let $|S|$ denote the number of elements in $S$, and let $n(S)$ be the number of subsets of $S$, including the empty set and the set $S$ itself. If $A$, $B$, and $C$ are sets for which $n(A)+n(B)+n(C)=n(A\cup B\cup C)$ and $|A|=|B|=100$, then what is the minimum possible value of $|A\cap B\cap C|$?

(A) 96 (B) 97 (C) 98 (D) 99 (E) 100

Solution

$\fbox{B}$

See also

1991 AHSME (ProblemsAnswer KeyResources)
Preceded by
Problem 29
Followed by
Problem 30
1 2 3 4 5 6 7 8 9 10 11 12 13 14 15 16 17 18 19 20 21 22 23 24 25 26 27 28 29 30
All AHSME Problems and Solutions

The problems on this page are copyrighted by the Mathematical Association of America's American Mathematics Competitions. AMC logo.png